Union and Intersection of Sets

Click For Summary
The discussion revolves around finding the union and intersection of two sets, A and B, defined as A = {x ∈ R | |x| > 1} and B = {x ∈ R | -2 < x < 3}. The user initially describes A as all real numbers except for the interval between -1 and 1, while B is the interval from -2 to 3. The user concludes that the union A ∪ B equals all real numbers (R) and the intersection A ∩ B consists of the intervals (-2, -1) and (1, 3). The correctness of the notation and answers is confirmed by another participant, who also suggests using interval notation for clarity.
TranscendArcu
Messages
277
Reaction score
0

Homework Statement


Let A = {x\in R | |x| &gt;1}, B = {x\in R | -2&lt;x&lt;3}. Find A \cup B and A\cap B

The Attempt at a Solution

I thought I might attempt this via a number line. Since I don't know how to make a number line in Latex, I'll describe it. I have A as being all of R except for the region bounded from -1 < x < 1. I have B as the region bounded from -2<x<3.

I then observed, A \cup B = R and A \cap B = { x \in R | -2&lt;x&lt;-1, 1&lt;x&lt;3}. But I'm sure if I have used the correct notation or if these answers are even correct.
 
Physics news on Phys.org
TranscendArcu said:

Homework Statement


Let A = {x\in R | |x| &gt;1}, B = {x\in R | -2&lt;x&lt;3}. Find A \cup B and A\cap B

The Attempt at a Solution

I thought I might attempt this via a number line. Since I don't know how to make a number line in Latex, I'll describe it. I have A as being all of R except for the region bounded from -1 < x < 1. I have B as the region bounded from -2<x<3.

I then observed, A \cup B = R and A \cap B = { x \in R | -2&lt;x&lt;-1, 1&lt;x&lt;3}. But I'm (not?) sure if I have used the correct notation or if these answers are even correct.
To make the braces, { , } , show in LaTeX, use the backslash, \ , character with each brace: \{ , \} .

I take it that you mean:
" Let \text{A}=\{x\in \mathbb{R} | |x| &gt;1\},\ \text{B}=\{x\in \mathbb{R} | -2&lt;x&lt;3\}. Find \text{ A}\cup\text{B} and \text{A}\cap\text{B}\,. "


Then your answers are correct.
\text{ A}\cup\text{B} = \mathbb{R}

\text{A}\cap\text{B} = \{ x \in \mathbb{R} | -2&lt;x&lt;-1, 1&lt;x&lt;3\}​

Of course you can write them in interval notation as
\text{ A}\cup\text{B} =(-\infty,\,\infty)

\text{A}\cap\text{B} =(-2,\,-1)\cup(1,\,3)​

.
 
Question: A clock's minute hand has length 4 and its hour hand has length 3. What is the distance between the tips at the moment when it is increasing most rapidly?(Putnam Exam Question) Answer: Making assumption that both the hands moves at constant angular velocities, the answer is ## \sqrt{7} .## But don't you think this assumption is somewhat doubtful and wrong?

Similar threads

  • · Replies 6 ·
Replies
6
Views
2K
  • · Replies 1 ·
Replies
1
Views
2K
  • · Replies 2 ·
Replies
2
Views
1K
Replies
3
Views
2K
  • · Replies 6 ·
Replies
6
Views
2K
  • · Replies 8 ·
Replies
8
Views
2K
  • · Replies 3 ·
Replies
3
Views
956
  • · Replies 6 ·
Replies
6
Views
4K
  • · Replies 12 ·
Replies
12
Views
2K
  • · Replies 9 ·
Replies
9
Views
2K